ebook img

Non local Poincaré inequalities on Lie groups with polynomial volume growth PDF

0.2 MB·English
Save to my drive
Quick download
Download
Most books are stored in the elastic cloud where traffic is expensive. For this reason, we have a limit on daily download.

Preview Non local Poincaré inequalities on Lie groups with polynomial volume growth

NON LOCAL POINCARE´ INEQUALITIES ON LIE GROUPS WITH POLYNOMIAL VOLUME GROWTH 0 EMMANUEL RUSS AND YANNICK SIRE 1 0 2 n a Abstract. Let G be a realconnectedLie groupwith polynomial J volume growth, endowed with its Haar measure dx. Given a C2 2 positive function M on G, we give a sufficient condition for an L2 2 Poincar´e inequality with respect to the measure M(x)dx to hold ] onG. Wethenestablishanon-localPoincar´einequalityonGwith A respect to M(x)dx. F . h Contents t a m 1. Introduction 1 [ 2. A proof of the Poincar´e inequality for dµ 5 M 1 3. Proof of Theorem 1.4 7 v 3.1. Rewriting the improved Poincar´e inequality 8 5 7 3.2. Off-diagonal L2 estimates for the resolvent of LM 8 0 α/4 3.3. Control of L f and conclusion of the proof of 4 M . L2(G,dµM) 1 Theo(cid:13)rem 1.4(cid:13) 10 (cid:13) (cid:13) 0 4. Appendix A:(cid:13)Techni(cid:13)cal lemma 16 0 1 5. Appendix B: Estimates for gt 16 j : References 17 v i X r a 1. Introduction Let G be a unimodular connected Lie group endowed with a measure M(x)dx where M L1(G) and dx stands for the Haar measure on G. ∈ By “unimodular”, we mean that the Haar measure is left and right- invariant. We always assume that M = e v where v is a C2 function − on G. If we denote by the Lie algebra of G, we consider a family G X = X ,...,X 1 k { } of left-invariant vector fields on G satisfying the H¨ormander condition, i.e. istheLiealgebrageneratedbytheX s. AstandardmetriconG, calleGd the Carnot-Caratheodory metric, isi′naturally associated with X 1 2 EMMANUEL RUSSANDYANNICKSIRE and is defined as follows: let ℓ : [0,1] G be an absolutely continuous → path. We say that ℓ is admissible if there exist measurable functions a ,...,a : [0,1] C such that, for almost every t [0,1], one has 1 k → ∈ k ℓ(t) = a (t)X (ℓ(t)). ′ i i i=1 X If ℓ is admissible, its length is defined by 1 1 k 2 ℓ = a (t) 2dt . i | | | | Z0 i=1 ! X For all x,y G, define d(x,y) as the infimum of the lengths of all ∈ admissible paths joining x to y (such a curve exists by the H¨ormander condition). This distance is left-invariant. For short, we denote by x | | the distance between e, the neutral element of the group and x, so that the distance from x to y is equal to y 1x . − | | For all r > 0, denote by B(x,r) the open ball in G with respect to the Carnot-Caratheodory distance and by V(r) the Haar measure of any ball. There exists d N (called the local dimension of (G,X)) ∗ ∈ and 0 < c < C such that, for all r (0,1), ∈ crd V(r) Crd, ≤ ≤ see [NSW85]. When r > 1, two situations may occur (see [Gui73]): Either there exist c,C,D > 0 such that, for all r > 1, • crD V(r) CrD ≤ ≤ where D is called the dimension at infinity of the group (note that, contrary to d, D does not depend on X). The group is said to have polynomial volume growth. Or there exist c ,c ,C ,C > 0 such that, for all r > 1, 1 2 1 2 • c ec2r V(r) C eC2r 1 1 ≤ ≤ and the group is said to have exponential volume growth. When G has polynomial volume growth, it is plain to see that there exists C > 0 such that, for all r > 0, (1.1) V(2r) CV(r), ≤ which implies that there exist C > 0 and κ > 0 such that, for all r > 0 and all θ > 1, (1.2) V(θr) CθκV(r). ≤ NONLOCAL POINCARE´ INEQUALITIES 3 Denote by H1(G,dµ ) the Sobolev space of functions f L2(G,dµ ) M M ∈ such that X f L2(G,dµ ) for all 1 i k. We are interested in L2 i M ∈ ≤ ≤ Poincar´e inequalities for the measure dµ . In order to state sufficient M conditions for such an inequality to hold, we introduce the operator k L f = M 1 X MX f M − i i − Xi=1 n o for all f such that 1 f (L ) := g H1(G,dµ ); X MX f L2(G,dx), 1 i k . M M i i ∈ D ∈ √M ∈ ∀ ≤ ≤ (cid:26) (cid:27) n o One therefore has, for all f (L ) and g H1(G,dµ ), M M ∈ D ∈ k L f(x)g(x)dµ (x) = X f(x) X g(x)dµ (x). M M i i M · ZG i=1 ZG X In particular, the operator L is symmetric on L2(G,dµ ). M M Following [BBCG08], say that a C2 function W : G R is a Lyapunov → function if W(x) 1 for all x G and there exist constants θ > 0, ≥ ∈ b 0 and R > 0 such that, for all x G, ≥ ∈ (1.3) L W(x) θW(x)+b1 (x), M B(e,R) − ≤ − where, for all A G, 1 denotes the characteristic function of A. We A ⊂ first claim: Theorem 1.1. Assume that G is unimodular and that there exists a Lyapunov function W on G. Then, dµ satisfies the following L2 M Poincar´e inequality: there exists C > 0 such that, for all function f H1(G,dµ ) with f(x)dµ (x) = 0, ∈ M G M R k (1.4) f(x) 2dµ (x) C X f(x) 2dµ (x). M i M | | ≤ | | ZG i=1 ZG X Let us give, as a corollary, a sufficient condition on v for (1.4) to hold: Corollary 1.2. Assume that G is unimodular and there exist constants a (0,1), c > 0 and R > 0 such that, for all x G with x > R, ∈ ∈ | | k k (1.5) a X v(x) 2 X2v(x) c. | i | − i ≥ i=1 i=1 X X Then (1.4) holds. 4 EMMANUEL RUSSANDYANNICKSIRE Notice that, if (1.5) holds with a 0, 1 , then the Poincar´e inequal- ∈ 2 ity (1.4) has the following self-improvement: (cid:0) (cid:1) Proposition 1.3. Assume that G is unimodular and that there exist constants c > 0, R > 0 and ε (0,1) such that, for all x G, ∈ ∈ k k 1 ε (1.6) − X v(x) 2 X2v(x) c whenever x > R. 2 | i | − i ≥ | | i=1 i=1 X X Then there exists C > 0 such that, for all function f H1(G,dµ ) M ∈ such that f(x)dµ (x) = 0: G M (1.7) R k k X f(x) 2dµ (x) C f(x) 2 1+ X v(x) 2 dµ (x) i M i M | | ≥ | | | | i=1 ZG ZG i=1 ! X X We finally obtain a Poincar´e inequality for dµ involving a non local M term: Theorem1.4. LetG bea unimodularLiegroup with polynomialgrowth. Let dµ = Mdx be a measure absolutely continuous with respect to the M Haar measure on G where M = e v L1(G) and v C2(G). Assume − ∈ ∈ that there exist constants c > 0, R > 0 and ε (0,1) such that (1.6) ∈ holds. Let α (0,2). Then there exists λ (M) > 0 such that, for any α ∈ function f (G) satisfying f(x)dµ (x) = 0, ∈ D G M f(x) f(Ry) 2 (1.8) | − | dxdµ (y) λ (M) V ( y 1x ) y 1x α M ≥ α ZZG×G | − | | − | k f(x) 2 1+ X v(x) 2 dµ (x). i M | | | | Rn ! Z i=1 X Note that (1.8) is an improvement of (1.7) in terms of fractional non- local quantities. The proof follows the same line as the paper [MRS09] but we concentrate here on a more geometric context. InordertoproveTheorem1.4,weneedtointroducefractionalpowers of L . This is the object of the following developments. Since the M operator L is symmetric and non-negative on L2(G,dµ ), we can M M define the usual power Lβ for any β (0,1) by means of spectral ∈ theory. Section 2 is devoted to the proof of Theorem 1.1 and Corollary 1.2. Then, in Section 3, we check L2 “off-diagonal” estimates for the resol- vent of L and use them to establish Theorem 1.4. M NONLOCAL POINCARE´ INEQUALITIES 5 2. A proof of the Poincar´e inequality for dµ M We follow closely the approach of [BBCG08]. Recall first that the following L2 local Poincar´e inequality holds on G for the measure dx: forallR > 0, thereexistsC > 0such that, forallx G, allr (0,R), R ∈ ∈ all ball B := B(x,r) and all function f C (B), ∞ ∈ k (2.9) f(x) f 2dx C r2 X f(x) 2dx, B R i | − | ≤ | | ZB i=1 ZB X where f := 1 f(x)dx. In the Euclidean context, Poincar´e in- B V(r) B equalities for vector-fields satisfying H¨ormander conditions were ob- R tained by Jerison in [Jer86]. A proof of (2.9) in the case of unimodular Lie groups can be found in [SC95], but the idea goes back to [Var87]. A nice survey on this topic can be found in [HK00]. Notice that no global growth assumption on the volume of balls is required for (2.9) to hold. The proof of (1.4) relies on the following inequality: Lemma 2.1. For all function f H1(G,dµ ) on G, M ∈ k L W (2.10) M (x)f2(x)dµ (x) X f(x) 2dµ (x). M i M W ≤ | | ZG i=1 ZG X Proof: Assume first that f is compactly supported on G. Using the definition of L , one has M L W k f2 M (x)f2(x)dµ (x) = X (x) X W(x)dµ (x) M i i M W W · ZG i=1 ZG (cid:18) (cid:19) X k f = 2 (x)X f(x) X W(x)dµ (x) i i M W · i=1 ZG X k f2 (x) X W(x) 2dµ (x) − W2 | i | M i=1 ZG X k = X f(x) 2dµ (x) i M | | i=1 ZG X k 2 f X f X W (x)dµ (x) i i M − − W i=1 ZG(cid:12) (cid:12) kX (cid:12) (cid:12) (cid:12) (cid:12) X(cid:12)f(x) 2dµ (x).(cid:12) i M ≤ | | i=1 ZG X 6 EMMANUEL RUSSANDYANNICKSIRE Notice that all the previous integrals are finite because of the support condition on f. Now, if f is as in Lemma 2.1, consider a nondecreasing sequence of smooth compactly supported functions χ satisfying n 1 χ 1 and X χ 1 for all 1 i k. B(e,nR) n i n ≤ ≤ | | ≤ ≤ ≤ Applying (2.10) to fχ and letting n go to + yields the desired n ∞ conclusion, by use of the monotone convergence theorem in the left- hand side and the dominated convergence theorem in the right-hand side. Let us now establish (1.4). Let g be a smooth function on G and let f := g c on G where c is a constant to be chosen. By assumption − (1.3), (2.11) L W b f2(x)dµ (x) f2(x) M (x)dµ (x)+ f2(x) (x)dµ (x). M M M ≤ θW θW ZG ZG ZB(e,R) Lemma2.1showsthat(2.10)holds. Letusnowturntothesecondterm intheright-handsideof(2.11). Fixcsuchthat f(x)dµ (x) = 0. B(e,R) M By (2.9) applied to f on B(e,R) and the fact that M is bounded from R above and below on B(e,R), one has k f2(x)dµ (x) CR2 X f(x) 2dµ (x) M i M ≤ | | ZB(e,R) i=1 ZB(e,R) X where the constant C depends on R and M. Therefore, using the fact that W 1 on G, ≥ (2.12) k b f2(x) (x)dµ (x) CR2 X f(x) 2dµ (x) M i M θW ≤ | | ZB(e,R) i=1 ZB(e,R) X where the constant C depends on R,M,θ and b. Gathering (2.11), (2.10) and (2.12) yields k (g(x) c)2dµ (x) C X g(x) 2dµ (x), M i M − ≤ | | ZG i=1 ZG X which easily implies (1.4) for the function g (and the same dependence for the constant C). Proof of Corollary 1.2: according to Theorem 1.1, it is enough to find a Lyapunov function W. Define W(x) := eγ(v(x) infGv) − NONLOCAL POINCARE´ INEQUALITIES 7 where γ > 0 will be chosen later. Since k k L W(x) = γ X2v(x) (1 γ) X v(x) 2 W(x), − M i − − | i | ! i=1 i=1 X X W is a Lyapunov function for γ := 1 a because of the assumption on − v. Indeed, one can take θ = cγ and b = max L W + θW B(e,R) M − (recall that M is a C2 function). n o Let us now prove Proposition 1.3. Observe first that, since v is C2 on G and (1.6) holds, there exists α R such that, for all x G, ∈ ∈ k k 1 ε (2.13) − X v(x) 2 X2v(x) α. 2 | i | − i ≥ i=1 i=1 X X 1 Let f be as in the statement of Proposition 1.3 and let g := fM2. Since, for all 1 i k, ≤ ≤ 1 1 3 Xif = M−2Xig gM−2XiM. − 2 Assumption (2.13) yields two positive constants β,γ such that k (2.14) X f(x) 2(x)dµ (x) = i M | | i=1 ZG X k 1 X g(x) 2 + g2(x) X v(x) 2 +g(x)X g(x)X v(x) dx i i i i | | 4 | | i=1 ZG(cid:18) (cid:19) X k 1 1 = X g(x) 2 + g2(x) X v(x) 2 + X g2 (x)X v(x) dx i i i i | | 4 | | 2 i=1 ZG(cid:18) (cid:19) X (cid:0) (cid:1) k 1 1 g2(x) X v(x) 2 X2v(x) dx ≥ 4 | i | − 2 i i=1 ZG (cid:18) (cid:19) X k f2(x) β X v(x) 2 γ dµ (x). i M ≥ | | − i=1 ZG X (cid:0) (cid:1) Theconjunctionof(1.4), which holdsbecause of(1.6), and(2.14)yields the desired conclusion. 3. Proof of Theorem 1.4 We divide the proof into several steps. 8 EMMANUEL RUSSANDYANNICKSIRE 3.1. Rewriting the improved Poincar´e inequality. By the def- inition of L , the conclusion of Proposition 1.3 means, in terms of M operators in L2(G,dµ ), that, for some λ > 0, M (3.15) L λµ, M ≥ where µ is the multiplication operator by 1 + k X v 2. Using a i=1| i | functional calculus argument (see [Dav80], p. 110), one deduces from P (3.15) that, for any α (0,2), ∈ Lα/2 λα/2µα/2 M ≥ which implies, thanks to the fact Lα/2 = (Lα/4)2 and the symmetry of M M Lα/4 on L2(G,dµ ), that M M α/2 k f(x) 2 1+ X v(x) 2 dµ (x) i M | | | | ≤ ZG i=1 ! X 2 2 α/4 α/4 C L f(x) dµ (x) = C L f . M M M ZG(cid:12) (cid:12) (cid:13) (cid:13)L2(G,dµM) The conclusion o(cid:12)f Theorem(cid:12) 1.4 will follow(cid:13) by es(cid:13)timating the quantity (cid:12) (cid:12) (cid:13) (cid:13) Lα/4f 2 . L2(G,dµM) (cid:13) (cid:13) (cid:13)3.2. Off(cid:13)-diagonal L2 estimates for the resolvent of L . The M crucial estimates to derive the desired inequality are some L2 “off- diagonal” estimates for the resolvent of L , in the spirit of [Gaf59] . M This is the object of the following lemma. Lemma 3.1. There exists C with the following property: for all closed disjoint subsets E,F G with d(E,F) =: d > 0, all function f ⊂ ∈ L2(G,dµ ) supported in E and all t > 0, M (I+tL ) 1f + tL (I+tL ) 1f M − L2(F,dµM) M M − L2(F,dµM) ≤ (cid:13) (cid:13) (cid:13) (cid:13) (cid:13) (cid:13) 8e−C √dt k(cid:13)fkL2(E,dµM). (cid:13) Proof. We argue as in [AHL+02], Lemma 1.1. From the fact that L M is self-adjoint on L2(G,dµ ) we have M 1 (L µ) 1 k M − − kL2(G,dµM) ≤ dist(µ,Σ(L )) M where Σ(L ) denotes the spectrum of L , and µ Σ(L ). Then we M M M 6∈ deduce that (I+tL ) 1 is bounded with norm less than 1 for all t > 0, M − and it is clearly enough to argue when 0 < t < d. NONLOCAL POINCARE´ INEQUALITIES 9 In the following computations, we will make explicit the dependence of the measure dµ in terms of M for sake of clarity. Define u = M t (I+tL ) 1f, so that, for all function v H1(G,dµ ), M − M ∈ (3.16) u (x)v(x)M(x)dx+ t ZG k t X u (x) X v(x)M(x)dx = i t i · i=1 ZG X f(x)v(x)M(x)dx. ZG Fix now a nonnegative function η (G) vanishing on E. Since f ∈ D is supported in E, applying (3.16) with v = η2u (remember that t u H1(G,dµ )) yields t M ∈ k η2(x) u (x) 2 M(x)dx+t X u (x) X (η2u )M(x)dx = 0, t i t i t | | · ZG i=1 ZG X which implies k η2(x) u (x) 2 M(x)dx+t η2(x) X u (x) 2 M(x)dx t i t | | | | ZG ZG i=1 X k = 2t η(x)u (x)X η(x) X u (x)M(x)dx t i i t − · i=1 ZG X k t u (x) 2 X η(x) 2M(x)dx+ t i ≤ | | | | ZG i=1 X k t η2(x) X u (x) 2 M(x)dx, i t | | ZG i=1 X hence (3.17) k η2(x) u (x) 2 M(x)dx t u (x) 2 X η(x) 2 M(x)dx. t t i | | ≤ | | | | ZG ZG i=1 X Let ζ be a nonnegative smooth function on G such that ζ = 0 on E, so that η := eαζ 1 0 and η vanishes on E for some α > 0 to be − ≥ chosen. Choosing this particular η in (3.17) with α > 0 gives eαζ(x) 1 2 u (x) 2 M(x)dx t − | | ≤ ZG (cid:12) (cid:12) (cid:12) (cid:12) 10 EMMANUEL RUSSANDYANNICKSIRE k α2t u (x) 2 X ζ(x) 2 e2αζ(x)M(x)dx. t i | | | | ZG i=1 X Taking α = 1/(2√t max X ζ ), one obtains i i k k ∞ 1 eαζ(x) 1 2 u (x) 2 M(x)dx u (x) 2e2αζ(x)M(x)dx. t t − | | ≤ 4 | | ZG ZG Usin(cid:12)gthefact(cid:12)thatthenormof(I+tL ) 1 isboundedby1uniformly (cid:12) (cid:12) M − in t > 0, this gives eαζu eαζ 1 u + u t L2(G,dµM) ≤ − t L2(G,dµM) k tkL2(G,dµM) 1 (cid:13) (cid:13) (cid:13)(cid:0) eαζu (cid:1) (cid:13) + f , (cid:13) (cid:13) ≤ (cid:13)2 t L2(G(cid:13),dµM) k kL2(G,dµM) therefore (cid:13) (cid:13) (cid:13) (cid:13) eαζ(x) 2 u (x) 2 M(x)dx 4 f(x) 2 M(x)dx. t | | ≤ | | ZG ZG We choose(cid:12)now ζ(cid:12)such that ζ = 0 on E as before and additionnally that (cid:12) (cid:12) ζ = 1 on F. It can furthermore be chosen with max X ζ i=1,...k i k k ≤ C/d, which yields the desired conclusion for the L2 norm of (∞I + tL ) 1f withafactor4intheright-handside. SincetL (I+tL ) 1f = M − M M − f (I + tL ) 1f, the desired inequality with a factor 8 readily fol- M − − (cid:3) lows. 3.3. Control of Lα/4f and conclusion of the proof of M L2(G,dµM) Theorem 1.4. T(cid:13)his is n(cid:13)ow the heart of the proof to reach the conclu- (cid:13) (cid:13) sion of Theorem 1(cid:13).4. The(cid:13)following first lemma is a standard quadratic estimate on powers of subelliptic operators. It is based on spectral theory. Lemma 3.2. Let α (0,2). There exists C > 0 such that, for all ∈ f (L ), M ∈ D (3.18) 2 + Lα/4f C ∞t 1 α/2 tL (I+tL ) 1f 2 dt. (cid:13) M (cid:13)L2(G,dµM) ≤ 3 Z0 − − M M − L2(G,dµM) (cid:13) (cid:13) (cid:13) We n(cid:13)ow come to the desired estimate. (cid:13) (cid:13) (cid:13) (cid:13) Lemma 3.3. Let α (0,2) . There exists C > 0 such that, for all ∈ f (G), ∈ D ∞t 1 α/2 tL (I+tL ) 1f 2 dt − − M M − L2(G,dµM) ≤ Z0 (cid:13)(cid:13) f(x) f(y) 2 (cid:13)(cid:13) C | − | M(x) dxdy. V ( y 1x ) y 1x α ZZG×G | − | | − |

See more

The list of books you might like

Most books are stored in the elastic cloud where traffic is expensive. For this reason, we have a limit on daily download.